You are on page 1of 31

1

Solutions & Explanations


Section-A : JEE Advanced/ IIT-JEE
A

B
C

1.

cos(ln q)

2.

1
x - ,
2

5.
1.
1.
7.
13.
19.
25.
1.
7.
13.

f, {(1, 1)}
F
(a)
(b)
(c)
(d)
(d)
(c)
(d)
(a, b, c, d)

2.
2.
8.
14.
20.
26.
2.
8.
14.

F
(a)
(b)
(d)
(a)
(b)
(b, c)
(b)
(a, c)

2.

4.

11.

2x + 4 y - p = 0

1
1
0 , ; -, - 0,
2
2

3.
9.
15.
21.
27.
3.
9.

(c)
(a)
(d)
(d)
(a)
(c)
(b, d)

c-

6.

4.
10.
16.
22.
28.
4.
10.
1
4

(b)
(c)
(b)
(a)
(b)
(a)
(b, c)

ud

9.

3.

x0

4.

abe

5.
11.
17.
23.
29.
5.
11.

(d)
(b)
(a)
(c)
(c)
(a, b, c)
(a, b)

6.
12.
18.
24.
30.
6.
12.

(d)
(d)
(c)
(a)
(c)
(a, c)
(b, c, d)

10. (0, 0)

v2 - u2

3 3
12. l - , 0 0,
2 2

14. (0, 2)

2x + 4y + 3p = 0

7
5
and max. at x = 1

10
sq. units
3

3 3 2
r
4

f is min at x =

16.

19.

6 + p :6

3
2
20. x + x - x + 2

21. x + 2 y = 2 or x - 2 y = - 2

22.

x + y =1

23. b (-2, - 1) (1, )

1
-3
, c =3
24. a = - , b =
2
4

25.

4 3
sq. units
9

26. 2 kh

27.

28.

1
1
2
min at x = (b + b 2 - 1) , max at x = (b - b - 1)
4
4

31.

a=

35.

(2, 1)

1.

(A) p (B) r

1.

(c)

2.

(a)

3.

(a)

4.

(c)

5.

(b)

6.

(d)

7.

(c)

1.

(7)

2.

(0)

3.

(9)

4.

(1)

5.

(5)

6.

(9)

7.

(9)

15.

18.

y = ea ( x -1) ; 1 sq. unit

-2 a
29. ,
a 3

1
1
5
-5
,b=
, c = 2 , f ( x) = x2 - x + 2
4
4
4
4

32. xy = 1
42. 4 65

41. y = 2

43. 6

Section-B : JEE Main/ AIEEE


1.

(b) 2.

(a) 3.

(d) 4.

(a) 5.

(b) 6.

(d) 7.

(d) 8.

(a) 9.

(d) 10. (b) 11. (b) 12. (a)

13. (c) 14. (c) 15. (d) 16. (c) 17. (a) 18. (b) 19. (b) 20. (a) 21. (c) 22. (c) 23. (d)
24. (a) 25. (c) 26. (b) 27. (b) 28. (c) 29. (b) 30. (c) 31. (a)
Buy books : http://www.dishapublication.com/entrance-exams-books/engineering-exams.html

Then area of DPF1F2 is given by

FILL IN THE BLANKS :

1.

We have e-p / 2 < q < p / 2

A=

p
< ln q < ln p / 2
2
cos (p/2) < cos (ln q) < cos (ln p/2)
[Q cos is increasing in IV quad]
cos
(ln
q)
>
0
....... (1)

Also -1 cos q 1" q


\

ln (cos q) 0
From (1) and (2) we get, cos (ln q) > ln (cos q)
\ cos (ln q) is larger.
2.

3.

b sin q

ae

- ae

1
| -b sin q(ae + ae) |= abe | sin q |
2
| sin p | 1

\ Amax = abe
The given curve is C : y3 3xy + 2 = 0
Differentiating it with respect to x, we get

....... (2)

3 y2

dy
dy
- 3x - 3 y = 0
dx
dx

dy
y
=
dx - x + y 2
Slope of tangent to C at point (x1, y1) is
y1
dy
=
dx - x1 + y12

y = 2 x 2 - ln | x |

1 (2 x + 1) (2 x - 1)
dy
= 4x - =
dx
x
x
Critical points are 0, 1/2, 1/2

1 1
Clearly f (x) is increasing on - , 0 , and
2 2

dy
= 0 y1 = 0
dx
For y1 = 0 in C, we get no value of x1
\ There is no point on C at which tangent is horizontal
\ H= f

For horizontal tangent,

1
1
f (x) is decreasing on -, - 0, .
2 2

Let f (x) = log (1 + x) x for x > 1

dy 1
=
dx 0

1
-x
-1 =
1+ x
1+ x
We observe that,

For vertical tangent

f ( x) > 0 if 1 < x < 0 and f ( x) < 0 if x > 0


Therefore f increases in ( 1, 0) and decreases in (0, ).
Also f (0) = log1 - 0 = 0

From C, y13 - 3 y13 + 2 = 0

f ( x ) =

- x1 + y12 = 0 x1 = y12
y13 = 1 y1 = 1 x1 = 1
\ There is only one point (1, 1) at which vertical tangent
can be drawn
\ V = {(1, 1)}

x 0 f ( x) f (0)

log(1 + x ) - x 0 log(1 + x) x

TRUE / FALSE :

Thus we get , log (1 + x ) x, " x 0


4.

a cos q

5.

-1 ln(cos q) 0 " 0 < cos q 1

1
2

Let P(a cos q, b sin q) be any point on the ellipse


x2
a2

y2
b2

= 1 with foci F1 (ae, 0) and F2 (ae, 0)

P (acosq, bsinq)

1.

2.

If (x r) is a factor of f (x) repeated m times then


f ' (x) is a polynomial with (x r) as factor repeated (m 1)
times.
\ Statement is false.
Given that 0 < a < x.
Let f ( x ) = log a x + log x a = log a x +
Consider g ( y) = y +

F2 (ae, 0) O

F1(ae, 0)

1
log a x

1
, where loga x = y
y
2

1
1
Q y+ = y +22
y
y
But equality holds when y = 1
Buy books : http://www.dishapublication.com/entrance-exams-books/engineering-exams.html

3
\
\

x = a which is not possible.

1
> 2 gmin cannot be 2.
y

y+

\
\

f min cannot be 2.
Statement is false.

cos q
( x - a(cos q + q sin q))
sin q
y sin q a sin2 q + a sin q cos q
= x cos q + a cos2 q + a q sin q cos q
x cos q + y sin q = a
As q varies inclination is not constant.
\ (a) is not correct.
Clearly does not pass through (0, 0).

y a (sin q q cos q) = -

MCQ's WITH ONE CORRECT ANSWER :

1.

(a) Consider the function f (x) = ax3 + bx2 + cx on [0, 1]


then being a polynomial. It is continuous on [0, 1],
differentiable on (0, 1) and
f (0) = f (1) = 0
[as given a + b + c = 0]
\ By Rolle's theorem $ x (0,1) such that
f ( x ) = 0 3ax 2 + 2bx + c = 0
Thus equation 3ax2 + 2bx + c = 0 has at least one root
in [0, 1].

2.

It's distance from origin =


4.

y
a

5.

6.
For max/min, dA = 0
dx

d A
dx 2

2 d 2 - x2

f ( x) =
=

ln(p + x)
ln(e + x)

1
1
ln(p + x )

ln(e + x) (e + x )
p + x
[ln( e + x )]2
( e + x) ln(e + x) - (p + x ) ln(p + x)
(e + x)( p + x ) (ln( e + x)) 2

< 0 on (0, ) since 1 < e < p

A is max at x = d / 2 then y = d / 2
\ f (x) decreases on (0, ) .
D is isosceles for max. area.
8.
(b) Let y = x25 (1 x)75
dy
(c) dx = a(- sin q + sin q + q cos q) = a q cos q (1)
= 25 x 24 (1 - x )75 - 75 x 25 (1 - x )74

dq
dx
= 25x24 (1 x)74 (1 x 3x)
dy
= a(cos q - cos q + q sin q) = a q sin q (2)
= 25x24 (1 x)74 (1 4x)
dq
Dividing (2) by (1), we get
dy
For maximum value of y,
=0
dy
dx
(slope of tangent)
= tan q
dx
x = 0, 1, 1/4, x = 1/ 4 (0,1)
Buy books : http://www.dishapublication.com/entrance-exams-books/engineering-exams.html
\
\

3.

(b) We have f ( x) =

(-2 x).(d 2 - 2 x 2 )

-8d d
+0
.
d A
2 2
At x = d / 2,
=
= -ve
dx 2 4(d 2 - d 2 / 2)
2

f (x) is increasing on (-, -1) and decreasing on


(-1, -) .

7.

2 d -x
4(d 2 - x 2 )

dy
= 0 at x = 1 and 2
dx

a
+ 2bx + 1 = 0 or 2bx2 + x + a = 0
x
has 1 and 2 as its roots.
\ 2b 1 + a = 0
(1)
8b + 2 + a = 0
(2)
Solving (1) and (2) we get a = 2, b = 1/2.
(d) For y2 = 4ax, y-axis is tangent at (0, 0), while for
x2 = 4ay, x-axis is tangent at (0, 0). Thus the two curves
cut each other at right angles.
(d) f ( x) = - ( x + 2)e- x = -( x + 1) e- x = 0
x=1
For x (-, -1), f ( x) > 0 and for

=0 x= d/ 2

(-4 x ).2 d 2 - x 2 - 2

=a

x (-1, ), f ( x ) < 0

-2 x
1
1
d 2 - x2 + x
=0
2
2 2 d 2 - x2
d 2 - x2 - x2

cos q + sin 2 q

a
2

which is constant
(b) y = a ln x + bx2 + x
has its extremum values at x = 1 and 2
\

1
(a) Area of DABC, A = x d 2 - x 2
2

Slope of normal = cot q


Equation of normal is

9.

Also at x = 0, y = 0, at x = 1, y = 0, and
at x = 1/4, y > 0
\ Max. value of y occurs at x = 1/4
(a) Slope of tangent at (x, f (x)) is 2x + 1
f ' (x) = 2x + 1 f (x) = x2 + x + c
Also the curve passes through (1, 2)
\ f (1) = 2
2 = 1 + 1 + c c = 0, \ f (x) = x2 + x
\

Required area =

Since, If f (x) increasing on (p / 4, p / 2)

12.

0 ( x2 + x) dx

f ( x) =

sin 2 x
where sin2 x is always +ve, when 0 < x 1 . But to
check Nr., we again let
h (x) = sin x x cos x

h ( x ) = x sin x > 0 for 0 < x 1


h (x) is increasing
h (0) < h (x), when 0 < x 1
0 < sin x x cos x, when 0 < x 1
sin x x cos x > 0, when 0 < x 1
f ' (x) > 0, x (0,1]
f (x) is increasing on (0, 1]

x
,0 < x 1
sin x

sin x - x cos x

Again g ( x) =
g ( x) =

\
13.

14.

or

15.

1
3p
= tan (given)
4
f (3)

1
p p
= tan + = -1
2 4
f (3)

f (3) = 1
(d) It is clear from figure that at x = 0, f (x) is not
differentiable.
y
y=x
2

y=x
O

+2

16.

(b) Let f ( x ) = e x - 1 - x then f ( x) = e x - 1 > 0 for


\
\

x (0,1)
f (x) is an increasing function.
f ( x ) > f (0), " x (0,1)

p
p
ex -1- x > 0 ex > 1 + x
<x<
4
2
\ (a) does not hold.
Buy books : http://www.dishapublication.com/entrance-exams-books/engineering-exams.html

p < 4 x < 2p

f (x) has neither maximum nor minimum at x = 0.

f ( x) > 0 - sin 4 x > 0 sin 4 x < 0

dy
= f ( x )(3,4)
dx

1
1
=f ( x)(3,4)
f (3)

but -

Hence g (x) is decreasing when 0 < x 1 .


(b) We are given f (x) = sin4 x + cos4 x
f ( x) = 4sin x cos x - 4cos x sin x
= 4 sin x cos x (cos2 x sin2 x)
= 2. sin 2x cos 2x = sin 4x
Now for f (x) to be increasing function

Q ex > 0 " x R

Therefore, slope of normal

p (x) is decreasing, when 0 < x 1


p (0) > p (x) 0 > tan x x sec2 x
\ g ( x) < 0

e x ( x - 1) ( x - 2) dx

(d) Slope of tangent y = f (x) is

=-

p ( x ) = -2 x sec2 x tan x < 0 for 0 < x 1

p/2

so that f ( x) = cos x
Again from graph it is clear that f (x) is increasing on
(0, p / 2) but f ' (x) is decreasing on (0, p / 2)
R is wrong.

1 < x < 2,

tan x - x sec 2 x , when


0< x 1
tan 2 x

O
1

For decreasing function, f ' (x) < 0


ex (x 1) (x 2) < 0 (x 1) (x 2) < 0

x
tan x

p ( x ) = sec 2 x - sec2 x - x.2 sec x.sec x tan x

11.

S is correct.
To disprove R let us consider the counter example :
f (x) = sin x on (0, p / 2)

(c) f (x) =

Here tan2 x > 0 But to check Nr. we consider


p (x) = tan x x sec2 x

p/2

Y = cos x

(c) We have f (x) =

Y = sin x

x3 x 2
1 1 5
= + = + =
2
3 2 6
3

10.

p 4p 3p
=
>
4 8
8
It will increasing on (p / 4,3p / 8) .
(d) From graph it is clear that both sin x and cos x in the
interval (p / 2, p) are decreasing function.

5
(b) Let g (x) = log (1 + x) x
1
x
-1 = < 0, " x (0,1)
1+ x
1+ x
g (x) is decreasing on (0, 1) \ x > 0
g (x) < g (0)
log (1 + x) x < 0 log (1 + x) < x
(b) holds. Similarly it can be shown that (c) and (d) do
not hold.
f (x) = xex (1x)
f ' (x) = ex (1x) + (1 2x) x ex (1x)
= ex (1x) (2x2 x 1) = ex (1 x) (2x + 1) (x 1)

then g ( x) =

\
17. (a)

For y = 2, x 2 =

1
(0,1) for the rest of the parts critical point
2
0 (0, 1). It can be easily seen that functions in options
(b), (c) and (d) are continuous on [0, 1] and differentiable
in (0, 1).

1
2 - x , x < 1/ 2
Now for f ( x) =
2
1 - x , x 1/ 2

m (b ) = -

m (b ) =

1
1 + b2

1 1
f (1/ 2+ ) = -2 - = 0
2 2
\
\
\

1 + b2

> 0 also b 2 0 1 + b 2 1

p p
p p
p
3 x - , x - , whose length is .
2 2
6 6
3
21. (d) The given curve is y3 + 3x2 = 12y

Intercept on x-axis =
Intercept on y-axis =

3 cos q

1
sin q

Sum of intercepts = S = 3 3 sec q + cosec q


dS
For min. value of S ,
=0
dq
3 3 sec q tan q cosec q cot q = 0

3 3 sin q
2

cos q

cos q
2

sin q

3
3
= 0 3 3 sin q - cos q = 0

1
=

3 3 3
1
tan q =
dy 1
= tan p / 6 q = p / 6
For vertical tangents
= 4 - y2 = 0
3
dx 0
Buy books : http://www.dishapublication.com/entrance-exams-books/engineering-exams.html

3 y2

dy
2x
dy
dy
=
+ 6 x = 12

dx 4 - y 2
dx
dx

x2
+ y 2 = 1 at
27

3 x cos q
+ y.sin q = 1
9

1
1 + b2
Thus m (b) = (0 , 1]
20. (a) 3 sin x 4 sin3 x = sin 3x which increases for

f is not differentiable at 1/ 2 (0,1)


LMV is not applicable for this function in [0, 1]

(3 3 cos q,sin q), q (0, p / 2) is

4(1 + b 2 )

1-
f f (1/ 2+ )
2

23. (c) Equation of tangent to the ellipse

4 b 2 - 4(1 + b 2 )

For range of m (b) :

1-
Here f = -1 and
2

1 b + 1

[-(b + 1)] = 2
2 b + 2

-24 + 8
= -ve (not possible)
3

point

b +1
and y-intercept = (b + 1)
b+2
Given Ar (D) = 2

-D
, D being the discreminant.
4a

x = 4 3

\ Req. points are (4 / 3, 2) .


22. (a) There is only one function in option (a) whose critical

x-intercept =

b2 + 2b + 1 = 4 (b + 2) b2 + 6b + 9 = 0
(b + 3)2 = 0 b = 3
19. (d) f (x) = (1 + b2) x2 + 2bx + 1
It is a quadratic expression with coeff. of
x2 = 1 + b2 > 0.
\ f (x) represents an upward parabola whose min value is

24 - 8 16
=
3
3

For y = 2, x 2 =

1
1/2
\ f (x) is increasing on [1/2, 1]
18. (c) Tangent to y = x2 + bx b at (1, 1) is
y 1 = (2 + b) (x 1)
(b + 2) x y = b + 1

y = 2

tan 3 q =

6
24.

(a) f (x) = x3 + bx2 + cx + d, 0 < b2 < c

28.

\
25.

f ( x ) = 3x + 2bx + c
Discriminant = 4b2 12c = 4 (b2 3c) < 0
f ( x ) > 0 " x R

f (x) is strictly increasing " x R


(d) For Rolle's theorem in [a, b]
f (a) = f (b), ln [0, 1] f (0) = f (1) = 0
Q The function has to be continuous in [0, 1]

x 0+

log x

=0
x -a
Applying L' Hospital's Rule
lim

x 0

S = (1 - a ) x 2 + ax, a (0, 2)

1/ x

(b)

For C2,

= 0 lim

dy 2
=
dx y

dy
dy
= 1 = m1 '
= 1= m1 and dx

(1, - 2)
dx (1, 2)

x0

- xa
=0a >0
x 0 - ax -a-1
x 0 a
Let the polynominal be P (x) = ax2 + bx + c
Given P (0) = 0 and P (1) = 1 c = 0 and a + b = 1
a=1b
P (x) = (1 b) x2 + bx
P' (x) = 2 (1 b) x + b
Given P ( x) > 0, " x [0,1]
2 (1 b) x + b > 0
When x = 0, b > 0 and when x = 1, b < 2
0<b<2

27.

For C1,

f (0) = lim f ( x) = 0 lim xa log x = 0

lim

26.

(b) The given curves, are


C1 : y2 = 4x
...(1) and C2 : x2 + y2 6 x + 1 = 0...(2)
Solving (1) and (2) we get
x2 + 4x 6x + 1 = 0 x = 1 and y = 2 or 2
\ Points of intersection of the two curves are (1, 2)
and (1, 2).

dy 3 - x
dy
=
= 1 = m2
\
dx
y
dx (1, 2)

dy
= -1 = m2 '
and dx
(1, - 2)
Q m1 = m2 and m1 ' = m2 '

\ C1 and C2 touch each other at two points.

29.

(c)

(a) The equation of tangent to the curve y = ex at (c, ec) is


(1)
y - ec = ec ( x - c)
and equation of line joining (c 1, ec1) and (c +1, ec+1)
is

The given function is

ec +1 - ec -1
[ x - (c - 1)]
y - ec-1 =
(c + 1) - (c - 1)
ec (e - e -1 )
[ x - c + 1]
2
Subtracting equation (1) from (2), we get

y - ec -1 =

(2)

e - e- 1 - 2 c e - e- 1
ec - ec -1 = ec ( x - c)
+e

2
2

e - e -1
1 - e-1 -

2 2 - e - e -1

=
x-c =
e - e -1 - 2
e - e -1 - 2
2
=
-1

e + e-1 - 2
2 - (e - e-1 )

e+e
-1
+ ve
2
=
=
= - ve
-1
- ve
e-e
12
xc<0 x<c
\ The two lines meet on the left of line x = c.

(2 + x )3 , -3 < x -1
f ( x) =
23
-1 < x - 2
x ,
The graph of y = f (x) is as shown in the figure. From
graph, clearly, there is one local maximum (at x = 1)
and one local minima (at x = 0)
\ total number of local maxima or minima = 2.

30.

-1 u
(c) Given that g(u) = 2 tan (e ) -1
\ g(u) = 2 tan ( e - u ) -

-1
= 2cot ( eu ) -

p
2

p
-1 1 p
= 2 tan u e 2
2

p
p
p
-1
= 2 - tan ( eu ) 2
2
2
p
p
-1 u
= - 2 tan (e )
2
2
\ g is an odd function.

-1 u
= p - 2 tan (e ) -

= g(u)
Also g '(u) =

2eu
1 + e 2u

> 0 , " u ( , )

\ g is strictly increasing on (, ) .
Buy books : http://www.dishapublication.com/entrance-exams-books/engineering-exams.html

7
3.
1.

(c)

y
x=

(c) The given polynomial is

p /2

x
y=
c=3p/2

p P
x=

p ( x) = a0 + a1x 2 + a2 x 4 + + an x 2 n , x R

and 0 < a0 < a1 < a2 < < an


Here we observe that all coefficients of different powers

\
\

We

have

P ( x) = 2a1 x + 4a2 x3 + + 2nan x 2 n -1

clearly P (x) increases for all x > 0 and decreases for all
x < 0.
P' (x) has no. max. value and min. value at x = 0.
ALTERNATE SOLUTION 2
We have P' (x) = 2a1x + 4a2x3 + + 2nanx2n1

4.

P ( x) = 0 x = 0
P(x) = 2a1 + 12a 2 x 2 ++ 2n(2n - 1)a n x 2n - 2

2.

x y = 1(1) [ pt ( x , y ) lies on the curve]


Also differentiating the curve xy = 1 with respect to x
we get

5.

-a
b
We must have,

x
a
a
= + ve - = +ve = -ve
y
b
b

a and b are of opposite sign.


either a < 0 and b > 0 or a > 0 and b < 0.

h( x ) h( y )

(3)

Hence, h (x) h (1) = 0 0 = 0 " x 0


(a, b, c) We are given that
3x 2 + 2 x - 1,
f ( x) =
37 - x,

(2)

Now from eq. (1), x y > 0 x , y are of same sign

From (2) and (3), we get h (x) = 0, " x 0

x
a
=y
b

f ( g ( x )) f ( g ( y )) , where x y

[as h (x) [0, )]

Also equation of normal suggests, slope of normal

Also h( x ) 0 " x 0

dy
dy
y
y+ x = 0
=dx
dx
x

x'
Slope of normal =
y'

y'
It is clear from the graph that the curves y = tan x and
y = x intersect at P in (p, 3p / 2) .
Thus the smallest +ve root of tan x x = 0 lies in
(p, 3p / 2) .
(a) Since g is decreasing in [0, )
\ For x y,
g ( x) g ( y)
........ (1)
Also g(x), g(y) [0, ) and f is increasing from [0, )
to [0, ).
\ For g (x), g (y) [0, )

h is decreasing function from [0, ) to [0, )


\ h( x ) h(0) , " x 0
But h (0) = 0
(given)
\ h( x ) 0 " x 0
(2)

xy = 1 at the point ( x , y ), then

dy
- y
=
dx ( x , y )
x

such that g ( x) g ( y)

P ( x ) |x =0 = + ve as a1 > 0
\ P (x) has only one minimum at x = 0.
(b, c) Let the line ax + by + c = 0 be normal to the curve

x'

of x, i.e., a0 , a1 , a2 , , an are positive.


Also only even powers of x are involved.
P (x) can not have any max. value.
Moreover P (x) is minimum i.e., a0, when x = 0.
P (x) has only one minimum.
ALTERNATE SOLUTION 1

-1 x 2
2< x3

Then on [ 1, 2], f ( x) = 6 x + 12
For -1 x 2, -6 6 x 12
6 6 x + 12 24
f ( x) > 0, " x [-1, 2]
\ f is increasing on [ 1, 2]
Also f (x) being polynomial for x [-1, 2) U (2,3]
f (x) is cont. on [ 1, 3] except possibly at
At x = 2,
2
LHL = lim f (2 - h) = lim 3(2 - h) + 12(2 - h) - 1
h 0

h 0

= 35
RHL = lim f (2 + h) = lim 37 - (2 + h) = 35
h 0

h 0

and f (2) = 3.22 + 12.2 1 = 35


LHL = RHL = f (2)

Buy books : http://www.dishapublication.com/entrance-exams-books/engineering-exams.html

f (x) is continuous at x = 2
Hence f (x) is continuous on [ 1, 3]
Again at x = 2

9.

(b, d)

f (2 + h) - f (2)
37 - (2 + h) - 35
= lim
=1
h
h
h0
h0

RD = lim

LD = lim

h0

f (2) - f (2 - h)
h

35 - 3(2 - h ) 2 - 12(2 - h ) + 1
h
h 0

= lim

-3h 2 + 24h
= 24
h
h0
As LD RD
\ f ' (2) does not exist. Hence f (x) can not have max. value
at x = 2.
(a, c) We have
= lim

6.

h( x) = f ( x)[1 - 2 f ( x) + 3 f ( x))2 ]

dy
at x = 3.
dx

dy
dy
= ve and x > 3,
= +ve
dx
dx
Change is from ve to +ve, hence minimum at x = 3.
Again minimum and maximum occur alternately.
\ 2nd minimum is at x = 1
10. (b, c) Let f (x) = ax3 + bx2 + cx + d
Then, f (2) = 18 8a + 4b + 2c + d = 18 (1)
(2)
f (1) = 1 a + b + c + d = 1
f (x) has local max. at x = 1
3a 2b + c = 0
(3)
x < 3,

f ( x) has local min. at x = 0 b = 0

2
1

= 3 f ( x) ( f ( x))2 - f ( x) +
3
3

1
17
f ( x) = (19 x3 - 57 x + 34) f (0) =
4
2

Note that h( x) < 0 whenever f ( x) < 0 and h ( x) > 0

Also f ( x) =

whenever f ( x) > 0 , thus, h (x) increases (decreases)


whenever f (x) increases (decreases).
(d) We have, f ( x) =

x -1

f (-1) < 0, f (1) > 0 x = -1 is a point of local max.


and x = 1 is a point of local min. Distance between ( 1,

x2 + 1

( x + 1) ( x + 1)

2) and (1, f (1)), i.e. (1, 1) is 13 2 5

11.

For max/min f ( x) = 0 x = 0.
f ( x) =

4(1 - 3x2 )

(a, b) Q g (x) =

0 f (t ) dt

f is min at x = 0

Min value of f (x) is 0 1 = -1


0 +1
ALTERNATE SOLUTION

ex ,
0 x 1

x -1
g ( x ) = f ( x ) = 2 - e , 1 < x 2
x - e,
2< x3

g ( x ) = 0 e x-1 = 2 or x - e = 0

x - 1 = log 2 or x = e

( x2 + 1)3

, f ( x) |x= 0 = +ve

f (x) =

x2 -1
2

x +1

( x 2 + 1) - 2

For f (x) to be min

x +1

2
x2 + 1

if x2 + 1 is min.
And x2 + 1 is min at x = 0.
\
8.

57 2
( x - 1) > 0, "x > 1
4

Also f ( x) = 0 x = 1, 1

4x

f ( x) =

fmin =

0 -1
= -1
0 +1

(4)

Solving (1), (2), (3) and (4), we get

= 3 f ( x)[{ f ( x) - 1/ 3}2 + 2/ 9]

7.

dy
= f ( x) x(e x - 1) ( x - 1)( x - 2)3 ( x - 3)5 = 0
dx
Critical points are 0, 1, 2, 3. Consider change of sign of

=1-

2
2

x +1

should be max, which is so

x = 1 + ln 2 or e
ex ,
0 x 1

x -1
g ( x ) = -e , 1 < x 2
1,
2< x3

NOTE THIS STEP

(b) The maximum value of f (x) = cos x + cos ( 2 x ) is 2


\ g (1 + ln 2) = -2 and g (e) = 1 g ( x) has local max.
which occurs at x = 0. Also, there is no value of x for
at x = 1 + ln 2 and local min. at x = e.
which this value will be attained again.
Buy books : http://www.dishapublication.com/entrance-exams-books/engineering-exams.html

9
14. (a, c) Let L = 8x, B = 15x and y be the length of square cut off
from each corner. Then volume of box
= (8x 2y) (15x 2y)y
V = 120x2y 46xy2 + 4y3

dV
= 120x2 92xy + 12y2
dy

dV
= 0 at y = 5 for maximum value of V.
dy
[30x2 23xy + 3y2]y = 5 = 0
6x2 23x + 15 = 0

1
1

(1+ ln 2)

3
e

Now

5
6
For x = 3, sides are 45 and 24.

Graph of g'(x)

x = 3,

Also graph of g ( x) suggests, g (x) has local max. at x


= 1 and local min. at x = 2
12. (b, c, d)

1
We have, f ( x ) = x cos , x 1
x

SUBJECTIVE PROBLEMS :

1.

f ( x) =

1 1
1
f '( x) = cos + sin
x x
x

(a - c + x + c) (b - c + x + c)
x+c

(a - c)(b - c)
+ ( x + c) + a + b - 2c
x+c

lim f '( x) = cos 0 + (0) (some finite value)

lim f '( x) = 1

f ' ( x) =

1 1
1 1
1
Also f "( x) =
sin - 2 sin - 3 cos
2
x x
x x
x
x
-1

1
f "( x) = 3 cos < 0, " x [1, )
x
x

f '( x) is strictly decreasing in [1, )

f '( x) > lim f '( x)

f ( x + 2) - f ( x)
>1
( x + 2) - x

(a + x)(b + x)
, a, b > c, x > -c
(c + x)

- ( a - c)(b - c )
( x + c) 2

f ( x) = 0 x = -c (a - c)(b - c)

x = -c + (a - c )(b - c) [+ve sign is taken Q x > - c ]

Also f ( x) =

\
\

2(a - c)(b - c)
( x + c)3

f min =

(a - c)(b - c)
( a - c)(b - c)

+ (a - c )(b - c)

+(a - c) + (b - c)
= (a - c) + (b - c) + 2 (a - c)(b - c)

We have f ( x ) = et (t - 2)(t - 3)dt


0

2.

2
f (x) = ex .(x - 2)( x - 3)
f (x) = 0 x = 2, 3
2

> 0 for a, b > c and x > c

f (x) is least at x = c + (a - c)(b - c)

f ( x + 2) - f ( x ) > 2
13. (a, b, c, d)
x

+1

= ( a - c + b - c )2
Given that x and y are two real variables such that x > 0
and xy = 1.
To find the minimum value of x + y.
Let S = x + y

f (x) = ex .2 x( x 2 - 5 x + 6) + ex (2 x - 5)
f (2) = ve and f (3) = + ve
\ x = 2 is a point of local maxima
and x = 3 is a point of local minima
Also or x(2, 3) f (x)< 0
f is decreasing on (2, 3)
Also we observe
f (0) < 0 and f (1) > 0
\ There exists some C (0, 1) such that f (C) = 0
\ All the options are correct.

1
x

S = x+

1
dS
= 1- 2
dx
x

For minimum value of S ,

1-

(using xy = 1)

dS
=0
dx

= 0 x = 1
x2
Buy books : http://www.dishapublication.com/entrance-exams-books/engineering-exams.html

10
But x > 0, \ x = 1
2

d S

Now

dx 2

d 2S
dx 2

\ | f "( x) |< 1 hence the result in this case.


Combining all the three cases we get
| f ( x) |< 1, V x [0,1]

2
x3

4.

= 2 = + ve

Let y = x1 / x

x =1

S is minimum when x = 1

1
log x
x
Differentiating w.r.t. x we get

log y =

1
S min = 1 + = 2
1
We are given that

\
3.

f ( x) = x1/ x , x > 0

1
.x - 1.log x
1 dy x
dy y(1 - log x)
=

=
2
y dx
dx
x
x2

x [0,1],| f ( x) |< 1 and f (0) = f (1)


To prove that | f ( x) |< 1, V x [0,1]
Here f (x) is continuous on [0, 1], differentiable on (0, 1)
and f (0) = f (1)
\ By Rolle's thm.,
$c (0,1) such that f (c) = 0

For max/min value put

y (1 - log x )

(1)

Now there may be three cases for x [0,1]


(i) x = c (ii) x > c (iii) x < c
Case I : For x = c.
If x = c then f ( x) = 0 < 1 [from (1)]

x2

d2y
dx

As f ( x) is continuous on [c, x] and differentiable on (c, x)

By LMV f (a) =

[Q f (c) = 0]

Now, x, c [0,1] and x > c


\ xc<1
also | f ( x) | < 1, " x (given)

........ (i)

........ (ii)

| f (a) | < 1

x=e

y x 2 - 2 xy (1 - log x )

x4

- xy
=
x 4 x=e

dy

0,1 - log
=
x 0 at
=
x e
Using=
dx

f (x) - f (c)
where a (c, x )
x-c

f ( x) = ( x - c) f (a )

= 0 log x = 1 x = e

1
dy
(1 - log x) 2
d
y
dx
x
Also,
=
dx 2

Hence the result | f ( x) |< 1 is obtained in this case.


Case II : For x > c
Consider the interval [c, x].

dy
=0
dx

- e1/ e
= - ve
e3

\
\

y is max at x = e
e1/e is the max. value of f (x).

x1/ x < e1/ e , " x

Combining (i) and (ii), ( x - c) | f () | < 1

\ Put x = p, we get, p1/ p < e1/ e


Raising to the power pe on both sides we get

\ | f ( x) |< 1 . Hence the result in this case.


Case III : For x < c
Consder the interval [x, c].
As f ( x) is continuous on [x , c] and differentiable on (x, c)

p e < e p or e p > p e
Given that f (x) and g (x) are differentiable for x [0,1] such
that
f (0) = 2 ; f (1) = 6, g (0) = 0 ; g (1) = 2

5.

as x, c [0,1] and x < c

To show that $ c (0,1) such that f (c) = 2g (c)


Let us consider h (x) = f (x) 2g (x)
Then h (x) is continuous on [0, 1] and
differentiable on (0, 1)
Also
h (0) = f (0) 2g (0) = 2 2 0 = 2
h (1) = f (1) 2g (1) = 6 2 2 = 2
\
h (0) = h (1)
\ All the conditions of Rolle's theorem are satisfied for
h(x) on [0, 1]

0 < c x < 1 also | f (b) |< 1 as | f ( x) |< 1, V x

| (c - x) f (b) |< 1

By LMV for b ( x, c)
f (b) =

f (c) - f ( x)
c-x

f ( x) = -(c - x) f (b)

[Using f (c) = 0 ]
\

| f ( x) |=| (c - x) f (b) |

$ c (0,1) such that h(c ) = 0


f ' (c) 2g' (c) = 0 f ' (c) = 2g' (c)

Buy books : http://www.dishapublication.com/entrance-exams-books/engineering-exams.html

11
6.

(0, c), y = x2, 0 c 5 .


Any point on parabola is (x, x2)
Distance between (x, x2) and (0, 1) is

= ln ( x + x 2 + 1)
As x + x 2 + 1 1 for x 1

D = x 2 + ( x 2 - c) 2
To minimum D we consider
D2 = x4 (2c 1) x2 + c2

\ f ( x ) 0, " x 0
Hence f (x) is increasing function.

2c - 1
1

= x2 +c

2
4

Now for x 0 f ( x) f (0)

2c - 1
2c - 1
which is minimum when x = 0 x2 =
2
2
2

7.

Dmin = c -

1
4

(1)

To show that 27 ab 2 4c3 .


Let us consider the function f (x) = ax2 + b/x
b
then f ( x) = 2ax - 2 = 0
x
x3 = b/2a x = (b/2a)1/3
2b
\ f ( x) = 2a + 3
x

1 + x ln( x + x 2 + 1) - 1 + x 2 0

9.

b
c
x
"x > 0, a > 0, b > 0

Given ax 2 +

1 + x ln( x + x 2 + 1) 1 + x 2
Let the house of the swimmer be at B.
\ d (AB) = L km.
Let the swimmer land at C, on the shore and let d (AC) = x km
\ d (SC) =

x2 + d 2

b 1/ 3
2b
f = 2a + 2a = 6a > 0
b
2a

b
f is minimum at x =
2a

1/ 3

b
so is for x =
2a

b
a
2a

2/3

b
a + b
2a

C Lx

distance
speed
Time from S to B = time S to C + time from C to B
Now using time =

1/ 3

b
(b / 2a)1/ 3

27b3 2a
c3
.
8
b
To show

1/ 3

27 ab2 4c3 Hence proved.

1 + x ln ( x + x 2 + 1) 1 + x 2 for x 0
Consider f (x) = 1 + x ln ( x + x2 + 1) - 1 + x2 )
2

Here, f ( x ) = ln( x + x + 1) +

T=

x
1 + x2
x 2 + 1
x

L x
1 2
x + d2 + u
v v

1
1.2 x
1
+ 0u 2 x2 + d 2
v
For either maximum or minimum, f ' (x) = 0

f '( x) =

1
- =0
u x2 + d 2 v
v2 x2 = u2 (x2 + d2) x2 (v2 u2) = u2 d2

f '( x) =

x2 =

x=

x + x2 + 1

1 +

x2 + d 2 L - x
+
u
v

Hence we take, f ( x ) =

3b 2a
c

1/ 3
2 b
(b / 2a)
As a, b are +ve, cubing both sides we get

8.

x 2 + d 2 and d (CB) = L - x

As (1) is true " x

ln ( x + x 2 + 1) 0

u2d 2
v2 - u 2

ud
2

v - u2

(as x ve)

Buy books : http://www.dishapublication.com/entrance-exams-books/engineering-exams.html

12
2x
2
2
x + d .1 - x.
1
2 x2 + d 2
Now, f "( x) =
u
( x 2 + d 2 )2

- sin( x + y )
1
[For parallel line m1 = m2]
=1 + sin( x + y )
2
2sin (x + y) = 1 + sin (x + y)
sin (x + y) = 1
Thus, cos (x + y) = 0
Using equation of curve and above result, we get, y = 0
sin x = 1
x = np + (1)n p/2, n Z
x = p/2, 3p/2
which belong to the interval [ 2p, 2p]
Thus the points on curve at which tangets are parallel to
given line are (p/2, 0) and ( 3p/2, 0)
The equation of tangent at (p/2, 0) is
\

d2
1
d2

=
> 0, " x
u x 2 + d 2 .( x 2 + d 2 )
u ( x 2 + d 2 )3 / 2

\
10.

Since the tangent to given curve is parallel to x + 2y = 0

ud

f has minimum at x =

v2 - u2
Equation of the curve is given by
x

y=

...(1)

1 + x2
Differentiating with respect to x, we get

1
y - 0 = - ( x - p / 2)
2
2y = x + p/2
2x + 4y p = 0
The equation of tangent at (3p/2, 0) is

dy 1 + x 2 - x (2 x )
1 - x2
=
=
dx
(1 + x 2 )2
(1 + x 2 )2
1- x2

dy
Again let f ( x) =
=
2 2
dx
(1 + x )

Now, f ' (x) =

(1 + x 2 )2 ( -2 x) - (1 - x 2 )2(1 + x 2 )(2 x )
(1 + x 2 )4
2

(1 + x )(-2 x ) - (1 - x )2.2 x
2 3

(1 + x )

12.
=

x (2 x - 6)
(1 + x 2 )3

1
sin 2 x (3sin x + 2l )
2
So, from f ' (x) = 0, we get x = 0
or 3 sin x + 2l = 0
=

For the greatest value of slope, we have


f '( x) =

x (2 x 2 - 6)
(1 + x 2 )3

= 0 x = 0, 3

3
Also, f " (x) = cos 2x (3 sin x + 2l) + sin 2 x cos x
2

Again we find,
f "( x ) =

12 x 2 (3 - x 2 )
(1 + x 2 )4

6(1 - x 2 )

-3
sin x , we have
2
f " (x) = 3 sin x cos2 x = 2l cos2 x
Now, if 0 < x < p/2, then 3/2 < l < 0 and therefore
f " (x) > 0.
f (x) has one minimum for this value of l.
Also for x = 0, we have f " (0) = 2l < 0, That is
f (x) has a maximum at x = 0
Again if p/2 < x < 0, then 0 < l < 3/2 and
therefore f " (x) = 2l cos2 x < 0.
So that f (x) has a maximum.
Also for x = 0, f " (a) = 2l > 0 so that f (x) has a minimum.
Thus, for exactly one maximum and minimum value of f (x),l
must lie in the interval
3/2 < l < 0 or 0 < l < 3/2

Therefore, for l =

(1 + x 2 )3

3
16
Thus, second order derivative at x = 0 is negative and second

11.

f " (0) = 6 and

f "( 3) =

order derivative at x = 3 is positive.


Therefore, the tangent to the curve has maximum slope at
(0, 0).
Equation of given curve y = cos (x + y), -2p x 2p
Differentiating with respect to x,

dy
dy
= - sin( x + y). 1 +
dx
dx

[1 + sin( x + y )]

dy
= - sin( x + y )
dx

dy
sin( x + y )
=dx
1 + sin( x + y )

1
y - 0 = - ( x + 3p / 2)
2
2y = x 3p/2
2x + 4y + 3p = 0
Thus the required equations of tangents are
2x + 4y p = 0 and 2x + 4y + 3p = 0.
The given function is,
f (x) = sin3 x + l sin2 x for p/2 < x < p/2
\ f ' (x) = 3 sin2 x cos x + 2l sin x cos x

13.
...(1)

i.e., l (-3/ 2, 0) (0, 3/ 2).


ATQ, AEDF is a parallelogram.
Let AF = x = ED
and AE = y = FD

Buy books : http://www.dishapublication.com/entrance-exams-books/engineering-exams.html

13
14. The equation of given curve can be expressed as

C (r , r)

x2

y2
= 1 where 4 < a2 < 8
a2 4
Clearly it is the question of an ellipse

by
x

y
A
2

(p , p)

P (acosq, bsinq)
M

B
2
(q , q)

Then from similar D's CED and CAB, we have


CE CA
b- y b
=
or
=
ED AB
x
c
Let P denote the area of ||gm AFDE.

[using eq. (1)]

c
P = sin A(by - y 2 )
b

dP c
= sin A(b - 2 y ) = 0
dy b

b
d P c
and
= sin A(-2) = - ve
2
dy 2 b
Hence area is maximum when y = b/2 and its value is

1
1
b c
b
P = . b - sin A = bc sin A = .ar (DABC )

2 b
2
4
2
p2
1 2
Now, ar (DABC ) = q
2
r2

-p 1
q

1 2
q - p2
2
r 2 - p2

-p

Let us consider a point P (a cos q, 2 sin q) on the ellipse.


Let the distance of P (a cos q, 2 sin q) from (0, 2) is L.
Then, L2 = (a cos q 0)2 + (2 sin q + 2)2
Differentiating with respect to q, we have

d ( L2 )
=0
dq
cos q [2a2 sin q + 8 sin q + 8] = 0
Either cos q = 0
or (8 2a2) sin q + 8 = 0

4
p
or sin q = 2
2
a -4
Since a2 < 8 a2 4 < 4

q=

-r 1

Operating, R2 R2 R1 and R3 R3 R1, we get


p2

(0, 2)

d ( L2 )
= cos q [-2a 2 sin q + 8sin q + 8]
dq
For max. or min. value of L we should have

Above gives y =

Also

4
a -4

q+ p 0
p-r 0

q- p 1
1
= ( q + p)( p - r )
- (r + p ) 1
2

1
= ar (DABC )
2

> 1 sin q > 1 which is not possible

d 2 ( L2 )
2

= cos q[-2a 2 cos q + 8cos q]

dq
+ ( sin q ) [ 2a2 sin q + 8 sin q + 8 ]

1
= (q + p) ( p - r ) (q + r )
2
Thus max. area of ||gm AFDE is

p d 2 ( L2 )
,
= 0 - [16 - 2a2 ] = 2(a 2 - 8) < 0
2 d q2
as a2 < 8
\ L is max. at q = p/2 and the farthest point is (0, 2).
15. We have,

At q =

f ( x) = [2(t - 1)(t - 2)3 + (t - 1)2 3(t - 2)2 ]dt


1

Then using the theorem,

d y( x)
g(t )dt = g[y( x)]y '( x) - g[f( x)]f '( x)

(
x
)
f
dx

We get,
f ' (x) = 2 (x 1) (x 2)3 + 3 (x 1)2 (x 2)2
= (x 1) (x 2)2 (2x 4 + 3x 3)
Hence Proved.
= (x 1) (x 2)2 (5x 7)
Buy books : http://www.dishapublication.com/entrance-exams-books/engineering-exams.html

1
= ( p + q)(q + r )( p - r )
4

...(1)

1
Then P = 2DAEF = 2. xy sin A
2
c
= y. (b - y ) sin A
b

(a, 0)

14
For extreme values f ' (x) = 0 x = 1, 2, 7/5
Now, f " (x) = (x 2)2 (5x 7) + 2 (x 1) (x 2) (5x 7)
+ 5 (x 1) (x 2)2
At x = 1,
f " (x) = 1 ( 2) = 2 < 0
\ f is max. at x = 1
At x = 2 f " (x) = 0
\ f is neither maximum nor minimum at x = 2.
At x = 7/5

( x - 1)3
1 2

( x - 1)3 .1 dx
= 4- x

3
3 0
0

x
( x - 1) 4
= 4 - ( x - 1)3
12
3
0

2 9 18
7 7
=
>0
f "( x) = 5 - 1 - 2 = 5
5 5
5 25 25

16.

\ f (x) is minimum at x = 7/5.


We have y = x (x 1)2, 0 x 2
dy
= ( x - 1)2 + 2 x ( x - 1) = ( x - 1)(3 x - 1)
dx
dy
For max. or min.
=0
dx
(x 1) (3x 1) = 0
x = 1, 1/3
d2 y
dx 2

At x = 1,

= 3 x - 1 + 3( x - 1) = 6 x - 4

d2y
dx 2

At x = 1/ 3,

= 2( + ve) \ y is min. at x = 1

d2y
dx

= -2(-ve) \ y is max. at x = 1/3

= tan 2 x - 4sin 2 x / 2 =
=

11
4
Max value of y is = - 1 =
3 3
27
Min value of y is = 1 (1 1)2 = 0

y=2

A
1

C (2, 2)

B
2

- 4sin 2 x / 2

- 4 sin 2 x / 2

...(i)

cos 2 x / 2 cos 2 x (as both are +ve)


cos 2 x / 2
cos2 x

cos 2 x / 2
cos 2 x

cos 2 x / 2
4sin 2 x / 2
- 1 0
2
cos x

From eq. (1), we get f '( x) 0

-1 0

f (x) is an increasing function on (0, p/2) for x 0


f ( x) f (0)

Now the curve cuts the axis x at (0, 0) and (1, 0). When x
increases from 1 to 2, y also increases and is +ve.
When y = 2, x (x 1)2 = 2
x 3 2x2 + x 2 = 0
(x 2) (x2 + 1) = 0
x=2
Using max./min. values of y and points of intersection with
x-axis, we get the curve as in figure and shaded area is the
required area.
\ The required area
= Area of squareOBCD - y dx

cos 2 x

cos 2 x

y
D

4 sin 2 x / 2 cos 2 x / 2

sin 2 x

cos 2 x / 2
= 4sin 2 x / 2
- 1
2
cos x

As cos x is decreasing function


\ x/2 x
cos x / 2 cos x

17.

2 10
2 1 1
= 4 - - + = 4 - = sq.units.
3 3
3 12 12
Let f (x) = 2 sin x + tan x 3x
\ f ' (x) = 2 cos x + sec2 x 3
= (sec2 x 1) 2 (1 cos x)

2 sin x + tan x - 3 x 0
2 sin x + tan x 3 x. Hence Proved
ALTERNATE SOLUTION

Let f (x) = 2 sin x + tan x 3x on 0 x < p / 2


then f ' (x) = 2 cos x + sec2 x 3
and f " (x) = 2 sin x + 2 sec2 x tan x
= 2 sin x [sec3 x 1]
f "( x) 0
for 0 x < p / 2
f ' (x) is an increasing function on 0 x < p / 2.
\ For x 0, f '( x) f '(0)
f '( x) 0 or 0 x < p / 2
f (x) is an increasing function on 0 x < p / 2

\ For x 0, f ( x) f (0)

2sin x + tan x - 3x 0, 0 x < p / 2

2sin x + tan x 3 x, 0 x < p / 2

= 2 2 - x( x - 1)2 dx
0
Buy books : http://www.dishapublication.com/entrance-exams-books/engineering-exams.html

Hence proved

15
18. As QR || XY diameter through P is ^ QR.
A

P = 2 + x + 2y

2
Now, area of rectangle ABCD = xy
\

...(1)

2q

p x

2 2
Let l be the light transmitted by coloured glass per sq. m.
Then 3l will be the light transmitted by clear glass per sq. m.
Hence the area of light transmitted

and area of arch DCED =

r
O
r

q
X

p x 2
= 3l ( xy ) + l
2 2

Now area of DPQR is given by


1
A = QR . AP
2
But QR = 2.QA = 2r sin 2q
and PA = OA + OP = r cos 2q + r
1
\ A = .2r sin 2q.( r + r cos 2q)
2
= r2. 2 sin q cos q . 2 cos2 q = 4 r2 sin q cos3 q
dA
=0
For max. value of area,
dq
4r2 [cos4 q 3 sin2 q cos2 q] = 0
cos2 q(cos2 q 3 sin2 q) = 0
1
tan q =
q = 30
3
2
Also d A = 4r 2 [-4cos3 q sin q - 6sin q cos3 q
d q2

= 4r2 [10 sin q cos 3 q + 6 sin3


d2A
dq

q= 30

+ 6sin3 q cos q]
q cos q]

1 3 3
1 3
= 4r 2 -10. .
+ 6. .
2 8
8 2

-15 3 3 3
2 -12 3
= 4r
+
= 4r
= -ve
8
8
8
\ A is maximum at q = 30
And Amax = 4r2 sin 30 cos3 30
2

1 3 3 3 3 2
= 4r 2
=
r
2
8
4
19. Let ABCEDA be the window as shown in the figure and let
AB = x m
E
BC = y m
D

B
Then its perimeter including the base DC of arch

px 2
...... (2)
A = l 3xy +

Substituting the value of y from (1) in (2), we get

1
4 + p
A = l 3x P -
2
2

px 2
x +

3Px 3(4 + p) 2 px 2
x +
=l

4
8
2
\

dA
px
3P 3(4 + p)
=l
x+
dx
2
4
2

For A to be maximum dA = 01
dx

3P
2
x=
-p 12 + 3p)
+

4
2

x=

Also

3P
4
6P
x=

2 5p + 24
5p + 24

d2A

-3(4 + p) p
= l
+ <0
2
4

dx
2

6P
5p + 24
[Using value of P from (1)]
(5p + 24 12 3p) x = 12y
(2p + 12) x = 12y

A is max when x =

y p+6
=
x
6
\ The required ratio of breadth to length of the rectangle
= 6+ p:6
20. Let f (x) = ax3 + bx 2 + cx + d
ATQ, f (x) vanishes at x = 2
...(1)
8a + 4b 2c + d = 0
f ' (x) = 3ax2 + 2bx + c
Againg ATQ, f (x) has relative max./min at

1
3
px

= 2x + 2 y + m
f ' ( 1) = 0 = f ' (1/3)

2
Buy books : http://www.dishapublication.com/entrance-exams-books/engineering-exams.html

x = 1 and x =

16
3a 2b + c = 0
and a + 2b + 3c = 0

... (2)
... (3)

21.

ax 4 bx 3 cx 2

14
+
+
+ dx =

4
3
2

-1 3

a b c
a b c
14
4 + 3 + 2 + d - 4 - 3 + 2 - d = 3

1
-1 + 1 + 4t 2 -1 - 1 - 4t 2
= t, - - t
,
2t
4t
4t
2

y = t ,t +

1
4t 2

+1
AB = 2t + +

4t 2
2t
2

Z=

5
4t

9
2t 4

+8

= + ve also

Z is minimum at t =

For t =

22.

1
t=
2

1
2

1
2

d 2Z
dt 2

1
t =2

or -

= + ve

1
2

normal chord is (from (2)) x + 2 y = 2

normal chord is x - 2 y = - 2
2
The given curve is y = (1+ x)y + sin1 (sin2 x)
Here at x = 0, y = (1 + 0)y + sin 1 (0) y = 1
\ Point at which normal has been drawn is (0, 1).
For slope of normal we need to find dy/dx, and for that we
consider the curve as
dy du dv
=
+
dx dx dx
where
u = (1 + x)y
...(i)
and
v = sin1 (sin2 x)
`
...(ii)
Taking log on both sides of equation (i) we get
log u = y log (1+ x)

y=u+v

1 du
y
dy
=
+ log(1 + x ).
u dx 1 + x
dx

dv
1
.2sin x cos x
=
dx
1 - sin 4 x

dv
2sin x
=
.
dx
1 + sin 2 x
Thus, we get,

dy
dy
2 sin x
y
= (1 + x ) y
+ log(1 + x ) +
+
1
dx
x
dx

1 + sin 2 x

+
+ 3 + 4t
16t 4 4t 2
For shortest chord, we have to minimize Z, and for that
dZ
=0
dt

dt 2

1
1

Consider Z = AB 2 = 2t + +
+ 1
2

4t

2t

,-

du
dy
y
= (1 + x) y
+ log(1 + x)
1
dx
+
x
dx

Also v = sin1(sin2x)

1
1

Thus, B - t - , t 2 +
+ 1
2

2t
4t
\ Length of normal chord

d 2Z

+1

For t = -

1
(x - t)
2t
...(2)
x + 2ty = t + 2t3
This normal meets the curve again at point B which can be
obtained by solving (1) and (2) as follows :
Putting y = x2 in (2), we get
2t x2 + x (t + 2t3) = 0,
D = 1 + 8t (t + 2t3)
= 1 + 8t2 + 16t4 = (1+ 4t2)2

x=

t=

dt

dy
= 2 x from (1)]
[where
dx

y - t2 = -

d 2Z

b
7

+d =
3
3
...(4)
b + 3d = 7
From (1), (2), (3), (4) on solving, we get
a = 1, b = 1, c = 1, d = 2
\ The required cubic is x3 + x2 x + 2.
The given curve is y = x2
...(1)
Consider any point A (t, t2) on (1) at which normal chord
drawn is shortest.
Then eq. of normal to (1) at A (t, t2) is
1
y - t2 = (x - t)
dy
2
dx (t , t )

+ 8t = 0
4t
2t 3
1 6t2 + 32t6 = 0
32 (t2)3 6t2 1 = 0 (2t2 1) (16t4 + 8t2 + 1) = 0
1
t 2 = (leaving ve values of t2)
2

14
Also, f ( x) =
-1
3
1

dy
dx
2sin x

[1 - (1 + x ) y log(1 + x )]

= y (1 + x ) y -1 +

1 + sin 2 x

Buy books : http://www.dishapublication.com/entrance-exams-books/engineering-exams.html

17

dy
=
dx

y (1 + x) y -1 +

2sin x

1 + sin 2 x
1 - (1 + x ) y log(1 + x)

dy
\ Slope of normal = 1
=1 ,
dx (0,1)
\ Equation of normal to given curve at (0, 1) is
y 1 = 1 (x 0)
x + y = 1.

23.

y
R

b3 - b 2 + b - 1
, 0 x<1
- x3 + 3
We have, f ( x ) =
b + 3b + 2
2 x - 3,
1 x 3

We can see from definition of the function, that


f (1) = 2(1) 3 = 1
Also f (x) is increasing on [1, 3], f ' (x) being 2 > 0.
\ f (1) = 1 is the smallest value of f (x)
Again f ' (x) = 3x2 for x [0, 1] such that f ' (x) < 0
f (x) is decreasing on [0, 1]
\ For fixed value of b, its smallest occur when x 1
i.e., lim f (1 - h) = lim - (1 - h)3 +
h 0

h 0

= -1 +

b3 - b2 + b - 1

-1 +

-1

(b 2 + 1)(b - 1)

0
(b + 2)(b + 1)
b2 + 3b + 2
(b 1) (b + 1) (b +2) 0

+
b3 - b 2 + b - 1

dy
= 0and P (-2, 0) lies on curve

dx x =-2

3ax2 + 2bx + c]x = 2 = 0


...(1)
12 a 4 b + c = 0
and 8a + 4 b 2c + 5 = 0
...(2)
[ Q ( 2, 0) lies on curve]
Also the curve cuts the y-axis at Q
\ For x = 0, y = 5 \ Q (0, 5)
At Q gradient of the curve is 3

dy

= 3 3x2 + 2bx + c]x = 0 = 3


dx x = 0
...(3)
c=3
Solving (1), (2) and (3), we get
a = 1/2, b = 3/4 and c = 3.

Q
(1, 0)

y'
Subtracting (1) from (2) we get
(x 1)2 x2 = (r2 1)
r2
2
Substituting this value of x in (2), we get
2x + 1 = r2 1

r4
+ y2 = r 2
4

x = 1-

y = r 1=

r2
4

r2
r2
R 1 - , r 1 - point being above x-axis.

2
4

1
Area of DQRS = SQ ordinate of point R
2

1
r2
A = r r 12
4
A will be max. if A2 is max.

A2 =

r 4 r 2 r 4 r6
1 - = 4
4 4 16

Differentiating A2 w.r. to r, we get


dA2
3
= r3 - r5
dr
8

b (-2, -1) (1, ).


Given that y = ax3 + bx2 + cx + 5 touches the x-axis at P (2, 0)

O S

P
(1, 0)

b3 - b 2 + b - 1

b3 - b2 + b - 1
b 2 + 3b + 2

x'

b2 + 3b + 2

b2 + 3b + 2
As given that the smallest value of f (x) occur at x = 1
\ Any other smallest value f (1)

24.

25. The given circle is x2 + y2 = 1


...(1)
which intersect x-axis at P ( 1, 0) and Q (1, 0).
Let radius of circle with centre at Q (1, 0) be r, where r is
variable.
Then equation of this circle is,
(x 1)2 + y2 = r2
...(2)

2
For A2 to be max. dA = 0
dr

2 2
3
r 3 1 - r 2 = 0 r =
8
3
d 2 ( A2 )
dr 2

d 2 ( A2 )
dr

= 3r 2 -

r2=

15 4
r
8

8 15 64
= 3 - = -ve
3 8 9
8
3

A2 and hence A is max. when, r =

2 2

Buy books : http://www.dishapublication.com/entrance-exams-books/engineering-exams.html

18
4

12 2
1 2 2

4 3 16 3

Max. area =

26.

4
3 3

1
a
Slope of the tangent at (1, 1) is = a
dy
=a

dx (1,1)

4 3
sq. units.
9

x y
+ = 1 , so that it makes an intercept
a b
of a units on x-axis and b units on y-axis. As it passes through
the fixed point (h, k), therefore we must have
Let the given line be

b
P
O

k
h
=1b
a

b = ak
a-h
1
Now Area of DOPQ = A = ab
2
\

1 ak
A = a

2 a - h

or

A=

y = ek(x1)

dy

=k
dx (1,1)

dy
From (1) and (2), = a = k
dx 1,1
(1)

\ y = ea(x 1) which is the required curve.


Now the area bounded by the curve, y-axis and normal to
curve at (1, 1) is as shown the shaded region in the fig.

y
B

dA
=0
da

k a 2 - 2ah

= 0 a = 2h
2 (a - h)2

2
2
2
Also, d A = (2a - 2h)( a - h) - 2(a - h) ( -1)( a - 2ah)
da 2
(a - h ) 4

=
a = 2h

dy
= ke k ( x -1)
dx

P (1, 1)

k 2a(a - h) - a 2

=0
2 (a - h)2

d2A

[using (1)]

da

dy
dy
y
= ky dy = kdx
dx
dx
y
log | y | = kx + C | y | = ekx+c = ec.ekx

k a2

2 a - h

For min. value of A,

But we are given that

(h, k)

(1)

y = ec e kx y = Aekx
Where A is constant. As the curve passes through (1, 1)
\ 1 = Aek
A = ek

The normal to the curve at P is


a (y 1) + (x 1) = 0
First we consider the case when a 0
Slope of normal at P (1 , 1) is = -

1 64 1 512 = 16 - 32
-
9 27
4 9 16 27

=
=

27.

(2h3 + 2h) (0)


h

A is min. when a = 2h

Amin =

2
> 0,[Qh > 0]
h

a(y 1)+(x 1)=0


x

Req. area = ar (PBC)


= ar (OAPBCO) ar (OAPCO)
1

0 ynormal dx - 0 ycurvedx

0 - a ( x - 1) + 1 dx - 0 ea( x-1) dx

1
1

= - ( x - 1)2 + x - e a( x -1)
2a
0 a
0

1 1 1 -a
1
1
- + e = 1 + e-a 2a a a
a
2a
Now we consider the case when a = 0. Then normal at (1, 1)
becomes x 1 = 0 which is parallel to y-axis, therefore tangent
at (1, 1) should be parallel to x-axis. Thus
= 1+

k 4h2

= 2kh
2 h
Buy books : http://www.dishapublication.com/entrance-exams-books/engineering-exams.html

19

dy
=0
dx
(1,1)
Since

(3)

dy
y gives y = ek (x 1)
dx
(as in a 0 case)

ax
xe ,
29. Given that, f ( x) =
2
3
x + ax - x ,
Differentiating both sides, we have

dy
=k
dx
(1,1)

2aeax + a 2 x e ax ; x 0
f ( x) =
x>0
2a - 6 x;

(4)

From (3) and (4), we get k = 0 and required curve becomes y


=1

For critical points, we put f (x) = 0

2
x = - , if x 0
a

x=1

2 a
f ( x) increases on - ,
a 3
30. Let b a = t, where a + b = 4

1
f ( x) = lnx - bx + x 2 , x > 0, b 0
8
(1)

f ( x) = 0 16 x 2 - 8bx + 1 = 0 (for max. or min.)


1
(2)
\ x = b b 2 - 1
4

Above will give real values of x if b2 1 0 i.e. b 1 or


b -1 . But b is given to be +ve. Hence we choose b 1

1
1
If b = 1 then x = ; If b > 1 then x = b b 2 - 1

4
4

1
2

+2=

16 x - 1

8x
8 x2
r
Its sign will depend on N , 16x2 1 as 8x2 is +ve. We shall
consider its sign for x =

1
1
and x = b b 2 - 1

4
4

f (x) = 0 at x = 1/4
\

Neither max. nor min. as f "( x ) = 0

2
2
N r of f '' (x) = 16x2 1 = [b + b - 1] - 1
= +ve for b > 1 \ Minima

or

2 a
f (x) is +ve on - ,
a 3

In this case the required area


= shaded area in fig. = 1 sq. unit.

f ( x) = -

a/3
2/a
It is clear from number line that

1
f ( x) = - b + 2 x
8x

a
, if x > 0
3

y=1

28.

x>0

axeax + eax ,
x0
f ( x) =
2
x>0
1 + 2 ax - 3 x ,
Again differentiating both sides, we have

dy
= kek ( x -1)
dx

x0

N r of f ( x) = (b - b2 - 1)2 - 1
= ve for b > 1
\ Maxima

4-t
t+4
and b =
2
2
as given a < 2 and b > 2 t > 0

Now

a=
a

0
=

g ( x) dx +
4- t
2
0

0 g ( x) dx

g ( x) dx +

4+ t
2
0

g ( x ) dx = f(t ) [say]

4 - t 1
4 + t 1
NOTE THIS STEP
- + g
2 2
2 2

f (t ) = g

d
Using
dx

v( x)

u ( x) f (t ) dt = f [v( x)].v ( x) - f [u( x)].u ( x)

1 4+ t
4- t
g
- g

2
2 2
Since g (x) is an increasing function (given)
\ for x1 > x2 g(x1) > g (x2)
=

Here we have 4 + t > 4 - t


2 2

4+ t
4-t
g
> g
2
2

f (t ) =

1 (4 + t )
(4 - t )
g
-g
> 0 f(t ) > 0

2
2
2

Buy books : http://www.dishapublication.com/entrance-exams-books/engineering-exams.html

20
It can be clearly seen that in order to prove the result it is
sufficient to prove that | p (1) | 1
We know that,

Hence f (t) increase as t increases.

31.

g ( x) dx +

0 g ( x) dx increases as (b a) increases.

| p (1) | = lim

Applying R3 R3 - R1 - 2 R2 we get

h0

2ax 2ax - a 2ax + b + 1


f ( x) =

b +1

-1

| p(1+ h) | + | p(1) |
NOTE THIS STEP
| h|
h0

lim

[Using | x y] | x | + | y | ]
But | p(1) | | e0 - 1|

2ax 2ax - 1
2ax -1
=
1
b
b +1
b

[Using equation (2) for x = 1]

| p(1) | 0

[Using C2 C2 - C1 ]

But being absolute value, | p(1) | 0 .

f ( x) = 2ax + b
Integrating, we get , f (x) = ax2 + bx + C
where C is an arbitrary constant. Since f has a maximum at x
= 5/2,
f (5/ 2) = 0 5a + b = 0
(1)
Also f (0) = 2 C = 2
and f (1) = 1 a + b + c = 1
\ a+b=1
(2)
Solving (1) and (2) for a, b we get,
a = 1/4, b = 5/4
1 2 5
x - x + 2.
4
4
Equation of the tangent at point (x, y) on the curve is

Thus, f ( x) =
32.

dy
( X - x)
dx
This meets axes in
Y y=

dx
dy

A x - y
0 and B 0, y - x

dy '

dx

1
dx 1
dy
Mid-point of AB is x - y , y - x
dy 2
dx
2
We are given
1
dx
1
dy
x - y = x and y - x = ya

dy
2
2
dx

dy
dy
dx
= x = -y
dx
y
x
Intergrating both sides,

Thus we must have | p(1) | = 0


Also | p(1 + h) | | eh - 1 |
(Using eqn (2) for x = 1 + h)

| eh - 1|
=1
h 0 | h |

Thus | p (1) | lim

| p (1) | 1
|a1 + 2a2 + .. + nan | 1
Given that -1 p 1 .
Consider f (x) = 4x3 3x p = 0

or
34.

1 3
Now, f (1/ 2) = - - p = -1 - p 0 as (-1 p)
2 2
Also f (1) = 4 3 p = 1 p 0 as ( p 1)
\ f (x) has at least one real root between [1/2, 1].

Also f ( x) = 12 x 2 - 3 > 0 on [1/2, 1]


f is increasing on [1/2, 1]
f has only one real root between [1/2, 1]
To find the root, we observe f (x) contains 4x3 3x which is
multipe angle formula of cos 3q if we put x = cos q.
\ Let the req. root be cos q then,
4 cos3q 3 cos q p = 0
cos 3q = p 3q = cos1 p

1
q = cos-1 ( p)
3
\

Root is cos cos-1 ( p) .


3

dy
dy
= log y = - log x + c
x2 y 2
y
x
35. The given curve is
+
= 1 (an ellipse)
6
3
Put x = 1, y = 1,
log 1 = log 1 + c c = 0
Any parametric point on it is P( 6 cos q, 3 sin q) .
log y + log x = 0 log yx = 0 yx = e0 = 1
Its distance from line x + y = 7 is given by
Which is a rectangular hyperbola.
6 cos q + 3 sin q - 7
Given that,
D=
p (x) = a0 + a1 x + a2 x2 + + anxn (1)
2
and | p (x) | | ex1 1|, " x 0
(2)
dD
For min. value of D,
=0
To prove that,
dq
| a1 + 2a2 ++ n an | 1
Buy books : http://www.dishapublication.com/entrance-exams-books/engineering-exams.html

33.

p(1 + h) - p(1)
h

21

- 6 sin q + 3 cos q = 0

tan q = 1/ 2
2

cos q =

x 0 f ( x) f (0)

and sin q =

sin (tan x) x sin (tan 0) 0


sin (tan x) x 0
sin (tan x) x

3
3
\ Required point P is (2, 1)
ALTERNATE SOLUTION :

Any point on given ellipse

x2 y 2
+
= 1 is
6
3

P( 6 cos q, 3 sin q)
Distance of P from x + y = 7 will be minimum if measured
along the normal of ellipse at P, and then PN ^ given line.
(slope of normal at P) (slope of given line) = 1
dx
-
dy (

6 cos q, 3 sin q )

2 3 sin q
6 cos q

sin q =

= 1 tan q =

1
2

cos q =

2
3

f ( x) = cos (tan x)sec2 x - 1


=

cos(tan x ) - cos 2 x
cos 2 x

As given 2 (1 cos x ) < x2, x 0

x2
cos x > 1 2

1
1 - tan 2 x - cos 2 x
2
f ( x) >
cos2 x

sin 2 x 1
2cos2 x
=
cos 2 x
=
\

sin 2 x(cos 2 x)
2 cos 4 x

> 0, " x [0, p / 4)

f ( x) > 0 f ( x) is an increasing function.

f (a) + f (b)
]
2

Multiplying (1) and (2) we get


[ f (4)]2 - [ f (0)]2
= f (a) f (b); a, b (0, 4)
8
or [f (4)]2 [f (0)]2 = 8 f (a) f (b)
Hence Proved.
(ii) To prove
4

0 f (t ) dt = 2[af (a 2 ) + bf (b2 )] " 0 < a, b < 2


Let I =

0 f (t )dt

Let t = u2 also t 0 u 0
dt = 2u du as t 4 u 2
\

0 f (t ) dt = 0 f (u 2 ).2u du

Consider, F ( x) =

2
Similary, cos (tan x) > 1 - tan x
2

f (4) - f (0)
= f (a) , for a (0, 4) (1)
4-0
Again since f is continuous on [0, 4] by intermediate mean
value theorem, we get

such that f (m) =

3
\ Required point is (2, 1).
36. Given that 2 (1 cos x) < x2, x 0
To prove sin (tan x) x, x [0, p / 4).
Let us consider f (x) = sin (tan x) x

Hence proved.
37. Given that f is a differentiable function on [0, 4]
\ It will be continuous on [0, 4]
\ By Lagrange's mean value theorem, we get
NOTE THIS STEP

f (4) + f (0)
(2)
= f (b) for b (0, 4)
2
[If f (x) is continuous on [a, b] then $ m (a, b)

(-1) = -1

2y

=1
x ( 6 cos q, 3 sin q)

For x [0, p / 4) ,

(1)

0 f (u 2 ).2u du

Then clearly F (x) is differentiable and hence continuous on


[0, 2]
By LMV theorem, we get some, m (0, 2)

F -F
such that F (m) = (2) (0)
2-0
2

f (m ).2m =

F (m) =

0 f (u

) 2u du

(2) NOTE THIS STEP


2
Again by intermediate mean value theorem,
$ a, b such that 0 < a < m < b < 2
F (a) + F (b)
, as f is continuous on [0, 2]
2

Buy books : http://www.dishapublication.com/entrance-exams-books/engineering-exams.html

22
= 23 (46)101 23 (46)101 23 45 46 + 1035 46 = 0

F is continuous on [0, 2]

f (m2 ).2m =

1
100
F (45 ) = F (46) = 0

f (a2 )2a + f (b2 ).2b


2

f (m 2 ).2m = af (a 2 ) + bf (b2 )
From (2) and (3) we get
2

\
\ Rolle's theorem is applicable.
Hence, there must exist at least one root of F ( x) = 0

(3)

f (u 2 ) 2u du = 2[a f (a 2 ) + b f (b2 )]

where 0 < a, b < 2


4

where 0 < a, b < 2


38.

40.

f (t ) dt = 2[a f (a 2 ) + b f (b2 )]
(Using eqn (1))

We are given that,

f (x) = sin x + 2 x Hence Proved.

dP( x )
> P( x ), ", x 1 and P (1) = 0
dx

dP( x) - x
- e P (x) > 0
dx

d -x
e P( x) > 0

dx
ex P(x) is an increasing function.

-x

[Using P (1) = 0]

P ( x ) > 0, " x > 1

P ( x ) > 0, " x > 1


[Qe - x > 0]
39. We are given,
P(x) = 51x101 2323x100 45x + 1035
To show that at least one root of P (x) lies in (451/100, 46),
using Rolle's theorem, we consider antiderivative of P (x)
102
2323x101 45 x 2
i.e. F ( x ) = x
+ 1035 x
2
101
2
Then being a polynominal function F(x) is continuous and
differentiable.

Now, F (451/100 ) =

102
(45)100

f ( x ) = - sin x -

101
2
100
45.(45)

(2)

3 p p 1
p
f = p + 1 - + 1 = - > 0
2
2 2 4 2
Let x = p be the point at which the max. of f (x) occurs [NOTE
There will be only one max. point in [0, p/2]. Since f '(x) = 0 is
only once in the interval.]
Consider , x [0, p]
f ' (x) > 0 f (x) is an increasing function.
f (0) f ( x) [as 0 x]

f ( x) 0

..... (4)

+ 1035(45)100

Y = f (x)
Dec.

3
>0
p

3
3
and f (p / 2) = 2 - (p + 1) = -1 - < 0
(3)
p
p
Equation (1), (2) and (3) shows that.
There exists a certain value of x [0, p / 2] for which
f ' (x) = 0 and this point must be a point of maximum for
f (x) since the sign of f ' (x) changes from +ve to ve.
Also we can see that f (0) = 0 and

Inc.

45
=
(45)100 - 23 45(45)100
2
2
100
45.(45)

.... (1)

Also for x [ p, p / 2]
f ' (x) < 0 f (x) is decreasing function.
..... (5)
for x < p/2, f (x) > f (p/2) > 0
y

101
100
2323(45)

3
f ( x) = cos x + 2 - (2 x + 1)
p

Also f (0) = 3 -

" x > 1, e- x P( x ) > e-1P (1) = 0

3 x( x + 1)
p

6
< 0, " x [0, p / 2]
p
f ' (x) is a decreasing function.

dP( x)
- P( x) > 0
dx
Multiplying by ex, we get,

e- x

i.e. P (x) = 0 in the interval 45100 , 46

Let us consider,

+ 1035(45)100 = 0

x = p x = p/2

Hence from (4) and (5) we conclude that


(46)102 2323(46)101 45(46)2
And F (46) =
+ 1035(46)
f ( x) 0, "x [0, p / 2] .
2
101
2
Buy books : http://www.dishapublication.com/entrance-exams-books/engineering-exams.html

23
41. Given that,
43. g (x) = (f ' (x))2 + f '' (x) f (x) =

| f (x1) f (x2) | < (x1 x2)2, x1 , x2 R


Let x1 = x + h and x2 = x then we get
| f (x + h) f (x) | < h2
| f (x + h) f (x) | < | h |2

Let h (x) = f (x) f ' (x)


Then, f (x) = 0 has four roots namely a, a, b, e
where b < a < c and c < b < d.
And f ' (x) = 0 at three points k1, k2, k3
where a < k1 < a, a < k2 < b, b < k3 < e
[ Q Between any two roots of a polynomial function
f (x) = 0 there lies atleast one root of f ' (x) = 0]
\ There are atleast 7 roots of f (x) . f ' (x) = 0

f ( x + h) - f ( x)
< |h|
h

Taking limit as h 0 on both sides, we get

lim

h0

f ( x + h) - f ( x)
< d (a small +ve number)
h

There are atleast 6 roots of

| f '( x) | < d f ' (x) = 0

3a 2b + c = 0
p' (x) has min. at x = 1
\ p'' (1) = 0
6a + 2b = 0
Solving (i), (ii), (iii) and (iv), we get
From (iv), b = 3a
From (iii), 3a + 6a + c = 0 c = 9a

MATCH THE FOLLOWING :

1.

...... (i)
...... (ii)

(A) f (x) = x + sin x on (p/2, p/2)


f ' (x) = 1 + cos x
As 0 cos x 1 for x (p/2, p/2)
\ f ' (x) > 0 on (p/2, p/2)
(A) p
(B) f (x) = sec x f ' (x) = sec x tan x.
Clearly f ' (x) < 0 in (p/2, 0) and f ' (x) > 0 in (0, p/2)

p p
\ On , f (x) is neither increasing non decreasing.
2 2

...... (iii)

(B) r
COMPREHENSION BASED Q UESTIONS :

...... (iv)

1.

(c) For k = 0, line y = x meets y = 0, i.e., x-axis only at one


point.
For k < 0, y = kex meets y = x only once as shown in the
graph.

From (ii), a 3a 9a + d = 6 d = 11a 6


From (i), a 3a + 9a + 11a 6 = 10
16a = 16 a = 1

b = 3, c = 9, d = 5
\ p (x) = x3 3x2 9x + 5
p' (x) = 3x2 6x 9 = 0
3 (x + 1) (x 3) = 0
x = 1 is a point of max. (given)
and x = 3 is a point of min.
[ Q max. and min. occur alternatively]
\ points of local max. is ( 1, 10) and
local min. is (3, 22).
And distance between them is
= [3 - (-1) 2 ] + (-22 - 10)2
= 16 + 1024 = 1040 = 4 65

d
( f ( x) f '( x)) = 0
dx

i.e. of g (x) = 0

f (x) is a constant function. Let f (x) = k i.e., y = k


As f (x) passes through (1, 2) y = 2
\ Equation of tangent at (1, 2) is,
y 2 = 0 (x 1) i.e. y = 2
42. Let p (x) = ax3 + bx2 + cx + d
p ( 1) = 10
a + b c + d = 10
p (1) = 6
a+b+c+d=6
p (x) has max. at x = 1
\ p' ( 1) = 0

d
( f ( x) f '( x))
dx

y
y=x
O
x

x'

y = ke , k < 0
y'

2.

(a) Let f (x) = kex x


Now for f (x) = 0 to have only one root means the line y
= x must be tangential to the curve y = kex.
Let it be so at (x1, y1) then
dy
dy
=
1 = ke x1

dx curve1
dx curve 2

Buy books : http://www.dishapublication.com/entrance-exams-books/engineering-exams.html

24
1
also y1 = ke x1 and y1 = x1
k
x1 = 1 1 = ke k = 1/e
(a) Q For y = x to be tangent to the curve
y = kex, k = 1/e
\ For y = kex to meet y = x at two points we should
x
e1 =

3.

1
1
k 0, as k > 0.
e
e
(c) For the statement P
f(x) + 2x = 2 (1 + x2)
(1 x)2 sin2x + x2 + 2x = 2(1+ x2)
(1 x)2 sin2x = x2 2x + 1 + 1
(1 x)2 sin2x = (1 x)2 + 1
(1 x)2 cos2x = 1
Which is not possible for any real value of x.
\ P is not true.
Also let H(x) = 2f(x) + 1 2x (1 + x)
H(0) = 2f(0) + 1 0 = 1
and H(1) = 2f(1) + 1 4 = 3
H(x) has a solution in (0, 1)
\ Q is true.

7.

have k <

4.

5.

1
\ In 0, , g(x) < 0
4
x
e (f (x) f (x)) < 0
f (x) < f(x)

1.

(7) The given function is f ( x) = 2 x3 - 15 x 2 + 36 x - 48

and A = {x | x 2 + 20 9 x}

2(t - 1)

(b) We have g(x) =


- ln t f (t)dt, x (1, )
t+1

A = x | x 2 - 9 x + 20 0

A = { x | ( x - 4)( x - 5) 0}
A = [4, 5]

2( x - 1)

\ g '( x ) =
- ln x f ( x )
x+1

Here f(x) > 0 , " x (1, )

Also f ( x) = 6 x2 - 30 x + 36 = 6( x 2 - 5x + 6)

Also let h(x) =

2( x - 1)
ln x
x +1

1 -(x - 1)2
=
< 0, x (1, )
h(x) =
( x + 1)2 x ( x + 1)2 x
\ h(x) is decreasing function.
\ For x > 1
h(x) < h(1)
h(x) < 0 " x > 1
\ g(x) < 0 " x (1, )
\ g(x) is decreasing on (1, ).

= 6( x - 2)( x - 3)
Clearly " x A, f ( x ) > 0
\ f is strictly increasing function on A.
\ Maximum value of f on A

6.

Also g(0) = g(1) = 0


g(x) < 0, x (0, 1)
ex f(x) < 0
f(x) < 0, x (0, 1)
(c) g(x) = exf(x)
g(x) = exf (x) exf(x)
= ex (f (x) f(x))
1
As x = is point of local minima in [0, 1]
4
1
\ g(x) < 0 for x 0,
4
1
and g(x) > 0 for x , 1
4

(d) We have f (x) 2f (x) +f(x) ex


[f (x) f (x) ] [f (x) f (x) ] e x

e x f (x) e x f (x) e x f (x) e x f (x) 1

d x
d x
e f (x)
e f (x) 1
dx

dx

d x
e f (x) e x f (x) 1

dx

d d x

e f (x) 1
dx dx

Let g(x) = exf(x)


Then we have g (x) 1 > 0
So g is concave upward.

= f (5) = 2 53 - 15 52 + 36 5 - 48
= 250 375 + 180 48 = 430 423 = 7.
2.

4
3
2
(0) Let p( x ) = ax + bx + cx + dx + e

p ( x)
Now lim 1 + 2 = 2
x0
x
p( x )
lim
=1
x 0 x 2
p(0) = 0 e = 0

...(1)

Applying LHospitals rule to eq n (1), we get


p ( x)
= 1 p (0) = 0
x0 2 x
lim

d =0
Again applying L Hospitals rule, we get
p ( x)
lim
= 1 p (0) = 2
x0 2
2 c = 2 or c = 1

p( x) = ax 4 + bx 3 + x 2

p ( x ) = 4ax 3 + 3bx 2 + 2 x
Buy books : http://www.dishapublication.com/entrance-exams-books/engineering-exams.html

25
As p(x) has extremum at x =1 and 2
\

p (1) = 0 and p (2) = 0

4a + 3b + 2 = 0

...(i)

32a + 12b + 4 = 0 or 8a + 3b + 1 = 0

...(ii)

Solving eqs (i) and (ii) we get a =


\

(5) We have f(x) = x + x 2 - 1


- x + x2 - 1, x < -1

2
x - x + 1, - 1 x 0
=
2
x - x + 1, 0 < x < 1

2
x + x - 1, x 1
From graph of the function, we observe there are 5
points of local maximum or local minimum.

1
and b = -1
4

1 4
x - x3 + x2
4

p( x) =

So, that p (2) =


3.

5.

16
-8+ 4 = 0
4

(9) The equation of tangent to the curve


y = f (x) at the point P (x,y) is
Y y dy
dy
=
(Y y ) = 0
or ( X x )
X x dx
dx
X

2x - 1

We have f (x) = -2 x - 1
-2 x + 1

2x + 1

dy
= x3
dx

dy y
= x2
dx x

x dx = 1

ve

1
\ y. =
x

1
x2
+C
x 2 dx =
x
2

6.

x3
y=
+ Cx
2
1
+C C = 3 2
2

\ y=

x3 3 x
+
2
2

At x = 3, y =

27 9
=9
2 2

\ f (3) = 9.
4.

(1) We have ,
f (x) = 2010 ( x 2009)( x 2010)2 ( x 2011)3 ( x 2012)4

,
x < -1
, -1 x 0
,
,

0< x<1
x >1

1 -1
, , -1, 0 and 1
2 2
+ve ve +ve ve +ve

1
0
1
We observe at five points f (x) changes its sign
\ There are 5 points of local maximum or local minimum.
(9) Q p(x) has a local maximum at x = 1 and a local minimum
at x = 3 and p(x) is a real polynomial of least degree
\ Let p(x) = k (x 1)(x 3 ) = k(x2 4x + 3)

x3

p(x) = k
- 2 x 2 + 3 x + C
3

Given p(1) = 6 and p(3) = 2


4
k + C = 6 and 0 + C = 2 k = 3

3
\ p(x) = 3(x 1)(x 3) p(0) = 9

As f (1) = 1 At x = 1, y = 1
\ 1=

Critical pts are

I.F. = e

Alternative

dy
dy
Y = x
y
dx
dx

Its y-intercept = y x

7.

(9) Vertical line x = h, meets the ellipse

x2 y 2
+
= 1 at
4
3

- 3

3
P h,
4 - h 2 and Q h,
4 - h 2
2
2

By symmetry, tangents at P and Q will meet each other


at x-axis.
y

As f ( x) = ln g ( x)
P

g ( x) = e f ( x) g '( x) = e f ( x) . f '( x)
For max/min, g '( x) = 0 f '( x) = 0
Out of two points one should be a point of maxima and
other that of minima.
\ There is only one point of local maxima.

(h, 0)
Q

Buy books : http://www.dishapublication.com/entrance-exams-books/engineering-exams.html

26
\ D(h) is a decreasing function.

xh y 3
+
4 - h2 = 1
4
6

Tangent at P is

4
which meets x-axis at R , 0
h
Area of DPQR =
i.e., D(h) =

1
3 4 - h2
2

32

4-

3
4
\ D1 =
1
2
2

- h
h

3 (4 - h 2 )3 2
2
h

D2 =

4 - h 2 (h 2 + 2)
dD
<0
= - 3
2
dh

1.

(b) Distance of origin from (x, y) =


=

x2 + y 2

at
= -1
cos t -

b min

(a) Let f (x) =

5.

(b)

ax3 bx 2
+
+ cx f (0) = 0 and f (1)
3
2

f ( x ) = 2 x3 - 9ax 2 + 12a 2 x + 1

1 = (2 - 1)3 + D D = 0
\ f (x) = ( x 1)3

6.

y - a sin q = tan q( x - a - a cos q)

y cos q - a sin q cos q = x sin q - a sin q


- a sin q cos q

\ a 2 = 2a a = 2or a = 0
but a > 0, therefore, a = 2.

Given

dy
9
9
= 2 =2 y =
dx
y
2

dx
dy
= - a sin q and
= a cos q
dq
dq
dy
= - cot q.
dx
\ The slope of the normal at q = tan q
\ The equation of the normal at q is

As per question p 2 = q

dy
dy 9
= 18
=
dx
dx y

(d)

x = a or x = 2a.At x = a max . and at x = 2a min


\ p = a and q = 2a

y 2 = 18 x 2 y

f ( x) = 6( x - 1). Inegrating, we get

Inegrating again, we get f ( x ) = ( x - 1)3 + D


The curve passes through (2, 1)

6 x 2 - 18ax + 12a 2 = 0 x 2 - 3ax + 2a 2 = 0

(a)

9
8

\ f ( x ) = 3 x 2 - 6 x + 3 = 3( x - 1) 2

f '( x ) = 6 x 2 - 18ax + 12 a 2 ; f ''( x ) = 12 x - 18a


For max. or min.

4.

D1 - 8D2 = 45 36 = 9

Slope at (2, 1) = f (2) = c = 3


[Q slope of tangent at (2,1) is 3]

a b
2a + 3b + 6c
+ +c =
=0
3 2
6
Also f (x) is continuous and differentiable in [0, 1] and
[0, 1[. So by Rolles theorem, f (x) = 0.
i.e ax2 + bx + c = 0 has at least one root in [0, 1].

(d)

33 3 9
=
2 1
2

f ( x) = 3 x 2 - 6 x + c

3.

45
5
8

9 9
\ Required point is ,
8 2

=a+b
\ Maximum distance from origin = a + b
2.

Putting in y 2 = 18 x x =

at
a 2 + b2 - 2ab cos t - ;

a 2 + b 2 + 2ab

1
1
h 1 D max = D and Dmin = D(1)
2
2

x sin q - y cos q = a sin q


y = ( x - a ) tan q
7.

which always passes through (a, 0)


(d) Let us define a function

ax3 bx 2
+
+ cx
3
2
Buy books : http://www.dishapublication.com/entrance-exams-books/engineering-exams.html
f ( x) =

27
Being polynomial, it is continuous and differentiable,
also,
f (0) = 0 and f (1) =

a b
+ +c
3 2

11.

(b) Let f (x) = an x n + an -1 x n -1 + ........... + a1 x = 0


The other given equation,
nan x n -1 + (n 1) an -1 x n - 2 + ....+ a1 = 0 = f (x)

2a + 3b + 6c
= 0 (given)
6
\ f (0) = f (1)
\ f (x) satisfies all conditions of Rolles theorem
therefore f (x) = 0 has a root in (0, 1)

Given a1 0 f (0) = 0

f (1) =

8.

i.e. ax 2 + bx + c = 0 has at lease one root in (0, 1)


(a) Area of rectangle ABCD = 2a cos q
(2b sin q) = 2ab sin 2q
Y
(-a cos q, b sin q)
B

(a cos q, b sin q)
A

(d)

D
(a cos q, - b sin q)

x = a ( cos q + q sin q)

dx
= a ( - sin q + sin q + q cos q )
dq
dx
= aq cos q
dq

Hence f ( x ) has a positive root smaller than a.


12. (a)

1
x 2
1
+ is of the form y + where y + 2 and
y
2 x
y
equality holds for y = 1

f (x) =

1 2
x 2
+ f '( x ) = - 2 = 0
2 x
2 x

x2 = 4 or x = 2, 2;

.....(1)

dy
= a [ cos q - cos q + q sin q ]
dq
dy
= aq sin q
.....(2)

dq
From equations (1) and (2) we get
dy
= tan q Slope of normal = cot q
dx
Equation of normal at 'q ' is y a (sin q q cos q )
= cot q (x a (cos q + q sin q )

y sin q a sin 2 q + a q cos q sin q


= x cos q + a cos 2 q + a q sin q cos q
x cos q + y sin q = a

Clearly this is an equation of straight line which is at


a constant distance a from origin.
10. (b) Given that

4
x3

1 2
x sin q
2

13. (c) Area =

Maximum value of sinq is 1 at q =

p
2

p
1 2
x at sin q = 1, q =
2
2
14. (c) Using Lagrange's Mean Value Theorem
Let f (x) be a function defined on [a, b]
Amax =

then, f '(c) =

f (b ) - f ( a )
b-a

....(i)

c [a, b]

Given f (x) = logex \ f ' (x) =

equation (i) become


1 f (3) - f (1)
=
c
3-1

d 4 3
dv
= 50 cm3/min
pr = 50
dt
3
dt
dr
= 50
4pr
dt

f ''( x) =

f ''( x )] x = 2 = + ve f ( x ) has local min at x = 2.

y = a ( sin q - q cos q)

x
= 1 i.e.,
2

at x = 2
ALTERNATE SOLUTION :

Area of greatest rectangle is equal to 2ab


When sin 2q = 1 .

9.

\ By Rolls theorem f(x) = 0 has root between ( 0, a )

\ Min value of function occurs at

X
C
( - a cos q, - b sin q)

Again f (x) has root a, f (a ) = 0


\
f (0) = f (a)

1 log e 3 - log e 1 log e 3


=
=
c
2
2

2
c=
dr
50

1
loge 3 c = 2 log3e
cm/min (here r = 10+5)
dt = 4 p(15) 2 =
18p
Buy books : http://www.dishapublication.com/entrance-exams-books/engineering-exams.html

1
x

28
15. (d) Given f (x) = tan1 (sin x + cos x)
f '(x) =

1 + (sin x + cos x)

d2y

.(cos x - sin x )

dx 2 x = p

1 + (sin x + cos x) 2

18.

f '(x) =

p
p
and maxima at x =
3
3
(b) Let f (x) = x7 + 14x5 + 16x3 + 30x 560
f '(x) = 7x6 + 70x4 + 48x2 + 30 > 0, " x R
f is an increasing function on R

19.

2
sin ( x ), if x < 0
=
2
sin ( x ), if x 0
2
sin x , if x < 0
=
2
sin x , if x 0

\ p = cos q and q = sin q

2 x cos x 2 , if x < 0
\ (go f ) (x) =
2
2 x cos x , if x 0
Here we observe
L (go f ) (0) = 0 = R (go f ) (0)
go f is differentiable at x = 0
and (go f ) is continuous at x = 0

Then p + q = cos q + sin q


We know that
- a 2 + b2 a cos q + b sin q a 2 + b 2

\ 2 cos q + sin q 2
Hence max. value of p + q is 2
ALTERNATE SOLUTION :
Since, p and q are positive real numbers
p2 + q2 = 1 (Given)
Using AM GM
2

p + q
p 2 + q2 + 2 pq
2
pq
( pq) =
\
2
4
1 + 2 pq
pq 1 + 2pq 4pq
4
1 2pq
or,
2pq 1
1
1
pq
or,
pq
2
2
Now, (p + q)2 = p2 + q2 + 2pq
(p + q)2 1 + 2

17.

1
p+q 2
2

(a) Let y = x3 px + q dy = 3 x 2 p
dx
For

dy
p
= 0 3x2 p = 0 x =
dx
3

d2y
dx 2

= 6x

The curve y = f (x) crosses x-axis only once.


\ f (x) = 0 has exactly one real root.
(b) Given that f (x) = x | x | and g (x) = sin x
So that
go f (x) = g ( f (x)) = g (x | x | )
= sin x | x |

p p
Hence, f (x) is increasing when n - ,
2 4
(c) Given that p2 + q2 = 1

= ve

1 + (sin x + cos x ) 2
if f (x) > O then f (x) is increasing function.
p
p p
Hence f (x) is increasing, if - < x + <
2
4 2
3p
p
<x<
4
4

16.

dx 2 x = p

Also lim f ( x) = and lim f ( x ) =

1 + (sin x + cos x) 2
p

2 cos x +

d2y

\ y has minima at x =

p
p

cos .cos x - sin .sin x


4
4

= + ve and

1
2.
cos x sin x
2

20.

\
(a)

2 cos x 2 + 4 x 2 sin x 2 , x < 0


Now (go f )'' (x) =
2
2
2
2 cos x - 4 x sin x , x 0
Here
L(go f )'' (0) = 2 and R (go f )'' (0) = 2
L(go f)'' (0) R (go f )'' (0)
go f (x) is not twice differentiable at x = 0.
Statement - 1 is true but statement -2 is false.
We have P (x) = x4 + ax3 + bx2 + cx + d
P' (x) = 4 x3 + 3ax2 + 2bx + c
But P' (0) = 0 c = 0
P(x) = x4 + ax3 + bx2 + d
As given that P ( 1) < P (a)
1a+b +d < 1+a+b+d
a>0
Now P ' (x) = 4x3 + 3ax2 +2bx
= x (4x2 + 3ax + 2b)
As P' (x) = 0, there is only one solution x = 0,
therefore 4x2 + 3ax + 2b = 0 should not have any real
roots i.e. D < 0

9a 2
>0
32
Hence a, b > 0 P' (x) = 4 x 3 + 3ax2 + 2bx > 0
"x > 0

9a2 32 b < 0

b>

Buy books : http://www.dishapublication.com/entrance-exams-books/engineering-exams.html

29
\
\

P (x) is an increasing function on (0,1)


P (0) < P (a)
Similarly we can prove P (x) is decreasing on ( 1, 0)
\ P ( 1) > P (0)
So we can conclude that
Max P (x) = P (1) and Min P (x) = P (0)
P(1) is not minimum but P (1) is the maximum of P.
21. (c) Since tangent is parallel to x-axis,

=
x

(t 2,

y2 = x
t)
x

8
dy
= 0 1- 3 = 0 x = 2 y = 3
\
dx
x
Equation of tangent is y 3 = 0 (x 2) y = 3
22. (c)
k - 2 x, if x -1
f ( x) =
2 x + 3, if x > -1

2x + 3

k-2x
1

So shortest distance between them is

25. (c)
This is true where k = 1
23. (d)

f ( x) =

e + 2e

f '( x ) =

-x

(e

f '( x) = x sin x , f '( x) = 0


x = 0 or sin x = 0

e
e

2x

+2

(e 2 x + 2) e x - 2e 2 x .e x
2x

+ 2)

5p
x = 2 p, p 0,

f ''( x) = x cos x +

f '( x) = 0 e 2 x + 2 = 2e 2 x

ex = 2

e2 x = 2

2
1
=
maximum f (x) =
4
2 2

0 < f ( x)

1
2 2

"x R

1
1
Since 0 < <
for some c R
3 2 2

2 x

1
2 x

sin x

(2 x cos x + sin x)

At x = p, f ''( x ) < 0
Hence, local maxima at x = p
At x = 2p , f ''( x) > 0
Hence local minima at x = 2p
26. (b) Y y =

dy
( X - x)
dx
Y

1
3
(a) Shortest distance between two curve occurred along
the common normal
Slope of normal to y2 = x at point P(t2, t) is 2t and
slope of line y x = 1 is 1.
As they are perpendicular to each other
\ ( 2t ) = 1

t=

f ( c) =

24.

3 2
8

B (0, y-xdy/dx)

(x, y)

1
2

1 1
\ P ,
4 2

1 1
1
and shortest distance = 2 4
2

A (x y)/(dy/dx), 0)
Y

X-intercept = x -

y
dy / dx

Buy books : http://www.dishapublication.com/entrance-exams-books/engineering-exams.html

30
Differentiating both the sides, w.r.t 't' we get,

xdy
dx
According to given statement

Y-intercept = y

x-

dV
dr
= 4pr 2
dt
dt

y
xdy
= 2 x and y = 2y
dy
dx

-y
=x
dy
dx

- xdy
=y
dx

and

4 3
pr
3
r3 = 3 243 = 3 35 = 36 = (32)3 r = 9

V = 972 p m3

dx dy

+
=0
x
y

lny = -lnc + lnc

Also, we have

c
x
Since the above line passes through the point (2,3)
\ c=6
y=

6
Hence y = is the required equation.
x

27. (b)

29.

1
+ 2bx + a
x
At x = 1, f(x) = 1 2b + a =0
a 2b = 1
\ f '( x) =

At x = 2,

tan x
>1
x

f(x) =

a + 4b = -

y = tan x

Thus, f ' ( x ) =
y=x
O

...(i)

1
+ 4b + a = 0
2

1
2

...(ii)

1
1
On solving (i) and (ii) we get a = , b = 2
4

In left neighbourhood of 0
tan x < x, tan x > 1 as ( tan x < 0)
x

1 x 1 2 - x2 + x
- + =
2x
x 2 2

2
- ( x + 1)( x - 2 )
-x2 + x + 2 - x - x - 2
=
=
=
2x
2x
2x

28.

dV
= 72 p
dt

(b) Given, f ( x ) = ln x + bx 2 + ax

In right neighbourhood of 0

\ 972 p =

dr 2
dr
=
72 p = 4 p 9 9
dt 9
dt

tan x
, x0

f ( x) = x
1, x = 0

tan x > x,

dV
= 72 p
dt
After 49 min, Volume = (4500 49 72)p
= (4500 3528)p
= 972 p m3

Now, it is given that

+
1

So maxima at x = 1,2
Hence both the statements are true and statement 2 is
at x = 0, f ( x) = 1
a correct explanation for 1.
30.
(c)
Equation of a line passing through (x1,y1) having
x = 0 is the point of minima
slope
m is given by y y1 = m (xx1)
So Statement 1 is true. Statement 2 obvious.
Since the line PQ is passing through (1,2) therefore its
4
equation is
(c) Volume of spherical balloon = V = pr 3
3
(y 2) = m (x 1)
where
m is the slope of the line PQ.
4pr 3
3
Now,
point
P (x,0) will also satisfy the equation of PQ
4500 p =
(Q Given, volume = 4500pm )
3
\ y 2 = m (x 1)
0 2 = m (x 1)
Buy books : http://www.dishapublication.com/entrance-exams-books/engineering-exams.html

31
2 = m (x 1)

x1=

x=

-2
m

-2
+1
m

Also, OP = ( x - 0 ) 2 + ( 0 - 0 )2 = x = -2 + 1
m
Similarly, point Q (0,y) will satisfy equation of PQ
\ y 2 = m (x 1)
y 2 = m (1) y = 2 m and OQ = y = 2m
Area of DPOQ =

1
1
2
(OP)( OQ) = 1 - ( 2 - m)
2
2 m
1
(Q Area of D = base height )
2

1
4
1
4

2 - m - + 2 = 4 - m +
2
m
2
m

=2-

m 2
2 m

Let Area = f (m) = 2 -

Now, f '' ( m) =
m= 2

-4
m3

=-

1
<0
2

1
>0
2
Area will be least at m = 2
Hence, slope of PQ is 2.
f '' ( m)

m = -2

31. (a) Since, y =

t dt , x R
0

dy
= x
dx
dy
=2
dx
x=2

But from y = 2x,


|x|=2

(1,2)

Points y =

m2 = 4 m = 2

Put f (m) = 0

therefore

-1 2
+
2 m2

Now, f ' ( m) =

f '' ( m)

m 2
2 m

t dt = 2

\ equation of tangent is
y 2 = 2(x 2) or y + 2 = 2(x + 2)
x-intercept = 1.

Buy books : http://www.dishapublication.com/entrance-exams-books/engineering-exams.html

You might also like